Tournament Integer!

m m is a positive integer such that m m ÷ m 2 \overline{mm} \div m^2 is equal to some integer, k k . What is the sum of all possible values of k k ?

Details and assumptions :

a a \overline{aa} means the number obtained by writing a a twice. For example if a = 103 a=103 , then a a = 103103 \overline{aa}=103103 .


This problem appeared in the TOT 2004.


This problem is from the set "Olympiads and Contests Around the World -1". You can see the rest of the problems here .


The answer is 18.

This section requires Javascript.
You are seeing this because something didn't load right. We suggest you, (a) try refreshing the page, (b) enabling javascript if it is disabled on your browser and, finally, (c) loading the non-javascript version of this page . We're sorry about the hassle.

2 solutions

First, after all, this is an awesome problem, thanks for compiling it , Mursalin!

Suppose m m is a positive integer with n n digits.

m ˉ m ˉ \bar{m}\bar{m} can be written as m ( 1 0 n + 1 ) m(10^{n} + 1)

Thus, m ( 1 0 n + 1 ) m 2 = k \frac{m(10^{n} + 1)}{m^{2}} = k

1 0 n + 1 m = k \frac{10^{n} + 1}{m} = k --- Equation (1)

For n = 1 n = 1 , we can clearly see that m = { 1 , 11 } m = \{1, 11\} , but the only acceptable m m is 1 1 because m m must be a one-digit integer in this case. So, if m = 1 m = 1 , that leads to k = 11 \boxed{k = 11}

For n = 2 n = 2 , we can clearly see that m = { 1 , 101 } m = \{1, 101\} , since 101 101 is a prime number. But there are no acceptable m m s, because m m must be a two-digit integer.

For n = 3 n = 3 , trial and error shows that 1001 = 7 11 13 1001 = 7 \cdot 11 \cdot 13 . This shows that m = { 1 , 7 , 11 , 13 , 77 , 91 , 143 , 1001 } m=\{1,7,11,13,77,91,143,1001\} . But the only acceptable m m is 143 143 because m m must be a three-digit integer in this case. So if m = 143 m = 143 , that leads to k = 7 \boxed{k = 7}

For n 4 n \geq 4 , we want to find the lowest upper bound of k k to limit the boundaries of k k , by doing that we must maximize k k by minimizing m m . The minimum value of an n n -digit integer is 1000...0 ( n n digits) = 1 0 n 1 10^{n-1} . So, the maximum value of k k is 1 0 n + 1 1 0 n 1 = 10 + 1 1 0 n 1 \frac{10^{n} + 1}{10^{n-1}} = 10 + \frac{1}{10^{n-1}} . And we know that 1 0 n 1 < 1 10^{n-1} < 1 , for all n 4 n \geq 4 . So,the boundaries of k k is [ 0 , 10 ] [0,10] , when k k is an integer.

Under the same circumstances, for n 4 n \geq 4 , we will rewrite the Equation (1) as 1 0 n + 1 k = m \frac{10^{n} + 1}{k} = m ---Equation (1)'. We will now then use the properties of parity to continue.

1 0 n + 1 10^{n} + 1 is always an odd integer. Thus k k must also be an odd integer for m m to be an integer. So, we cancel out the even integers and our used value 7 7 , and that leaves { 1 , 3 , 5 , 9 } \{1, 3, 5, 9\} . We cancel out 7 7 because even though it will work, it will just be a repeating value and that doesn't affect our answer.

For k = 1 k = 1 , n [ k 1 0 n + 1 ] \forall n[k | 10^{n} + 1] , substituting into the equation (1)', we will get m = 1 0 n + 1 m = 10^{n} + 1 , which has n + 1 n+1 digits. This contradicts the statement that m m has n n digits. So, goodbye 1 1

For 1 0 n + 1 10^{n} + 1 to be divisible by 3 3 , its digits' sum must be a multiple of 3 3 . However, the digits' sum is always 2 2 . So, goodbye 3 3 .

For 1 0 n + 1 10^{n} + 1 to be divisible by 5 5 , its last digit must be either 0 0 or 5 5 . However its last digit here is 1. So, goodbye 5 5 .

For 1 0 n + 1 10^{n} + 1 to be divisible by 9 9 , its digits' sum must be a multiple of 9 9 . However, the digits' sum is always 2 2 . So, goodbye 9 9 .

Thus, n 4 [ m 1 0 n + 1 ] \forall n \geq 4 [m \nmid 10^{n} + 1]

Thus, k = { 7 , 11 } k = \{7, 11\} (And this makes me think of 7-11 shops)

k s u m = 18 ! k_{sum} = \boxed{18}!

If there are any flaws, please let me know in the comments.

PS.: I'm stuck with this question for an hour, and I decided to turn on some music. Thanks to "Let it go" for cheering me up. XD

Your solution looks alright. But it's a little lengthy.

Notice that problem asks for k k , not m m . So you never have to worry about what m m is.

First rewrite the equation as you did and find the bound for k k . No matter what m m and n n are 1 < k 11 1<k\leq 11 .

Now eliminate 2 , 3 , 4 , 5 , 6 , 8 , 9 , 10 2, 3, 4, 5, 6, 8, 9, 10 and that leaves you with only 7 7 and 11 11 .

Show that both 7 7 and 11 11 are attainable. m = 1 m=1 and 143 143 works perfectly.

So our answer is 7 + 11 = 18 7+11=18 .

Mursalin Habib - 7 years ago

1000000001 is divisible by 7 too! The quotient is 142857143. Hence, the answer should be atleast 25. Note - I tried to use Excel to find out if higher values have any single divisors. It shows that till 10^15, only 11, 1001 and 1000000001 have single digit divisors

Vamsi Bethapudy - 7 years ago

Log in to reply

The only possible values of k k are 11 11 and 7 7 . It doesn't matter if 7 7 is attainable in multiple ways. The answer is still 18 18 .

Mursalin Habib - 7 years ago

At the bottom of your 5th paragraph, you bound k between 1 and 10 yet k can = 11, am I thinking wrong?

Trevor Arashiro - 6 years, 11 months ago

Log in to reply

in case of m = 1

Sanchayapol Lewgasamsarn - 6 years, 11 months ago
Lập Dị Ngài
Jan 17, 2015

For m=1 we have 11:1=k=11

For m>1 we have m has a digits, so 10^(a-1)<= m <10^a (1)

We also have mm=(10^a+1)m, so from the problem we have (10^a+1) is divided by m (2)

From (1) and (2) we have 1<k<10

Because (10^a+1) is not divided by 2,3,5, so k=7. This happens when a=3(mod6)

Sorry for the inconvenience but I don't know how to use Latex

0 pending reports

×

Problem Loading...

Note Loading...

Set Loading...